LSAT and Law School Admissions Forum

Get expert LSAT preparation and law school admissions advice from PowerScore Test Preparation.

 Administrator
PowerScore Staff
  • PowerScore Staff
  • Posts: 8917
  • Joined: Feb 02, 2011
|
#27120
Complete Question Explanation
(The complete setup for this game can be found here: lsat/viewtopic.php?t=11115)

The correct answer choice is (C)

From the discussion of inferences, when L is offered, then G, H, and Z must be offered, P and S cannot be offered, and M may or may not be offered. As this is a Could Be True question, immediately attack the uncertainty in this question: M. Only answer choice (C) addresses M, and as it could be true that M is not offered, answer choice (C) is correct.
 diwil
  • Posts: 9
  • Joined: May 19, 2019
|
#65334
Hello,

Thank you in advance for your help.

I got to the correct answer, (M not being offered/IN), but I am not quite convinced.
If L is Offered/IN, then G is also offered/in and P is Not offered/out. IF P is out then S is out. (I need S AND L to be out/not offered, in order to have M also not offered/out, right? That's the contrapositive of M -> L or S (first rule).
If L AND S MUST BE OUT (for M to be out), THEN I am not sure why M is out (because L is IN). I do have S out, but not L (L is In/being offered).
By process of elimination I ended up with M not being offered (letter C), but that would require my contrapositive to be:
IF L out OR S out, then M out. Right? However, my contrapositive is IF L out AND S out, then M out.

I am not sure if I made myself clear. Let me know if you understand my question, and thank you in advance!

Thank you for your help.
 Adam Tyson
PowerScore Staff
  • PowerScore Staff
  • Posts: 5153
  • Joined: Apr 14, 2011
|
#65375
Hey there diwil, I understand where you are coming from, and it looks like you are making a Mistaken Reversal of conditional logic.

When M is in, you need exactly one of L or S. Either L is in and S is out, or S is in and L is out. That means there are two ways to prove that M is out, using the contrapositive - if L and S are both OUT, or if L and S are both IN. Either of those would force M to be out. But, as it turns out, L and S can never both be in, as you correctly surmised due to what they each do to P.

So, M requires exactly one of L or S to be in. But having one of them in does not require that M be in - that is the mistaken reversal. M could be in, OR M could be out - it's optional at that point! The necessary condition (one of L or S) has been met, so the sufficient condition (M is in) MAY be met, but doesn't NEED to be met.

One last thing - this question isn't about what MUST be true, but about what COULD be true. It could be true that M is offered, and it could be true that M is not offered, because it is optional when L is offered. The other four answers cannot be true - S must be out, H and Z must be in, and there are at least 4 courses being offered (LGHZ).

Remember, when a necessary condition is met, the sufficient condition is just an option. Don't go backwards, stabbing yourself on the point of a conditional arrow, and conclude that the sufficient condition must occur! It's sufficient, but it isn't necessary!

Get the most out of your LSAT Prep Plus subscription.

Analyze and track your performance with our Testing and Analytics Package.